A biologist is studying the migratory patterns of sandhill cranes. His study focuses only on sandhill cranes that are bred and hatched in Alaska, and eventually migrate south to warmer climates. Each year he randomly tags a sample of 50 cranes with GPS trackers to gather data about their migratory patterns.

In this scenario, the population is
.

In this scenario, the sample is

Answers

Answer 1

Answer:

Population = All Sandhill cranes bred and hatched in Alaska that migrate south

Sample = The 50 crane samples which are tagged yearly.

Step-by-step explanation:

The population refers to all members or subjects which belongs to a defined study. Each and every subject which meets the requirement of a study makes up the population.

The sample however, is a subset of the population, the sample is a smaller group of subject which is selected from the population randomly to for a representative sample of the population. Here, it is the 50 sampled sandhill cranes bed per year.

Answer 2

Answer:

These are the answers

Step-by-step explanation:

A Biologist Is Studying The Migratory Patterns Of Sandhill Cranes. His Study Focuses Only On Sandhill

Related Questions

Suppose that g(x)= f(x)+ 6. Which statement best compares the graph of g(x) with the graph of f(x)?

A. The graph of g(x) is the graph of f(x) shifted 6 units down.

B. The graph of g(x) is the graph of f(x) shifted to the right.

C. The graph of g(x) is the graph of f(x) shifted 6 units to the left.

D. The graph of g(x) is the graph of f(x) shifted 6 units up.

Answers

Answer:

D

Step-by-step explanation:

The + 6 moves it up 6 units.

The correct answer is (D) "The graph of g(x) is the graph of f(x) shifted 6 units up."

What is the function?

A relationship between a group of inputs and one output is referred to as a function. In plain English, a function is an association between inputs in which each input is connected to precisely one output. A domain, codomain, or range exists for every function. Typically, f(x), where x is the input, is used to represent a function.

When we add a constant to a function, such as in the case of g(x) = f(x) + 6, it will shift the graph of f(x) upward by 6 units.

This is because, for any value of x, the value of f(x) will be added to 6, resulting in a vertical shift of the entire graph.

Option (A) is incorrect because adding 6 to f(x) would shift the graph up, not down.

Option (B) is incorrect because adding a constant to a function does not cause it to shift horizontally.

Option (C) is incorrect because adding 6 to f(x) would shift the graph right, not left.

D. The graph of g(x) is the graph of f(x) shifted 6 units up. Adding a constant term to a function will shift the graph of the function vertically. In this case, adding 6 to f(x) will shift the graph of f(x) upward by 6 units, resulting in the graph of g(x).

Learn more about function here:

https://brainly.com/question/29633660

#SPJ7

Find the distance between the pair of points: (0,1) and (1,0)

Answers

Answer:

sqrt(1^2 + 2^2)

[tex]\sqrt{2}[/tex]

Step-by-step explanation:

Which diagram represents the hypothesis of the converse of corresponding angles theorem?

Answers

Answer:

the first diagram

Step-by-step explanation:

first one

Rectangle ABCD translates 4 units down and 2 units to the right to form rectangle A'B'C'D'. The vertices of rectangle ABCD are labeled in alphabetical order going clockwise around the figure. If AB = 3 units and AD = 5 units, what is the length of B'C'?

Answers

Answer:

The length of BC is 14 units.

Step-by-step explanation:

[tex]hope \: \: it \: \: helps} \beta \alpha \infty [/tex]

The length of B'C' is 0 units.

What is translation?

It is the movement of the shape in left, right, up, and down direction.

The translated shape will have the same shape and shape.

There is a positive value when translated to the right and up.

There is a negative value when translated to the left and down.

We have,

The length of AD = 5 units.

Since the rectangle translates down by 4 units,

The length of A'D' =5 units.

The width of the original rectangle is AB, which is 3 units.

Since the rectangle translates to the right by 2 units,

The width of the new rectangle = 3 units.

Now,

The length of B'C' is the same as the length of AD', which is 5 units.

Subtracting 5 units from 5 units gives us a length of 0 units.

Thus,

The length of B'C' is 0 units.

Learn more about translation here:

https://brainly.com/question/12463306

#SPJ7

Need help finding the factor of 2y^2-2y-4

Answers

Answer:

hope it helps you............

Answer:

2(y - 2)(y + 1)

Step-by-step explanation:

Given

2y² - 2y - 4 ← factor out 2 from each term

= 2(y² - y - 2) ← factor the quadratic

Consider the factors of the constant term (- 2) which sum to give the coefficient of the y- term (- 1)

The factors are - 2 and + 1, since

- 2 × 1 = - 2 and - 2 + 1 = - 1 , then

y² - y - 2 = (y - 2)(y + 1)

Then

2y² - 2y - 4 = 2(y - 2)(y + 1) ← in factored form

Find the missing length indicated

Answers

x = 65

Step-by-step explanation:

cos theta = 25/x

cos theta = x/169

25/x = x/169

x² = 169 x 25

x = 65

The missing length x = 65, using the Pythagoras Theorem.

What is the Pythagoras Theorem?

According to the Pythagoras Theorem, in a right triangle, the square of the hypotenuse is equal to the sum of the squares of the legs.

How to solve the question?

In the question, we are asked to find the value of x.

In the right triangle ABC, by Pythagoras' Theorem,

AC² + BC² = AB²,

or, x² + BC² = (144 + 25)²,

or, BC² = 169² - x² ... (i).

In the right triangle ACD, by Pythagoras Theorem,

AD² + DC² = AC²,

or, 25² + DC² = x²,

or, DC² = x² - 25² ... (ii).

In the right triangle BCD, by Pythagoras Theorem,

BD² + DC² = BC²,

or, 144² + x² - 25² = 169² - x² {Substituting BC² = 169² - x² from (i) and DC² = x² - 25² from (ii)},

or, x² + x² = 169² + 25² - 144² {Rearranging},

or, 2x² = 28561 + 625 - 20736,

or, 2x² = 8450,

or, x² = 4225,

or, x = √4225 = 65.

Thus, the missing length x = 65, using the Pythagoras Theorem.

Learn more about the Pythagoras Theorem at

https://brainly.com/question/231802

#SPJ2

ASAP there are three marbles in a bag. One is red and two are black. What is the probability of picking a black marble first, putting it back in the bag and then picking a black marble? Use the following probability to find the answer.

Answers

Answer:

[tex] \frac{4}{9} [/tex]

Step-by-step explanation:

[tex]p = \frac{favorable \: outcomes}{total \: outcomes} = \frac{4}{9} [/tex]

Answer:  Choice D)  4/9

=============================================================

Explanation:

The probability you get a black marble on the first selection is 2/3 since we have 2 black marbles out of 2+1 = 3 total.

We put the marble back and then we have 2/3 as the probability of selecting another black marble on the second try. Nothing has changed because we put the marble back. That means the events are independent.

So we get (2/3)*(2/3) = 4/9 as the probability of selecting 2 black marbles in a row (with replacement).

The probability distribution for a random variable x is given in the table X: -5,-3,-2,0,2,3 Probability: .17,.13,.33,.16,.11,.10 Find the probability that X <_-3

Answers

Answer:

0.3 probability that [tex]x \leq -3[/tex]

Step-by-step explanation:

The probability distribution is given in the table.

Probability that x <= -3

The values that are -3 or lower are -3 and -5. So

[tex]P(X \leq -3) = P(X = -3) + P(X = -5)[/tex]

From the table:

[tex]P(X = -3) = 0.13, P(X = -5) = 0.17[/tex]. So

[tex]P(X \leq -3) = P(X = -3) + P(X = -5) = 0.13 + 0.17 = 0.3[/tex]

0.3 probability that [tex]x \leq -3[/tex]

Answer:0.3

Step-by-step explanation:

HELP ME PLEASE IF YOU DO YOU WILL GET BRAINLESS AND PLEASE EXPLAIN THE BEST YOU CAN

Answers

Answer:

<3=75°

Step-by-step explanation:

Angle 3 and angle 2x+95 are supplementary( supplementary angles add up to 180°)

So <3+2x+95=180

<3+2x=180-95

<3+2x=85( let's call this equation 1)

Next, angle 5 and angle 8x+71 are opposite angles (opposite angles are equal) therefore <5=8x+71

Now, <3 and <5 are co-interior angles(co-interior angles are supplementary)

So <3+8x+71=180

<3+8x=180-71=109

Thus, <3+8x=109(let's call this equation 2)

Now solving equation 1 and 2 simultaneously:

Make <3 the subject of equation 1

<3=85-2x

Put <3=85-2x into equation 2

85-2x+8x=109

6x=24

x=24/6=4

Now, remember that angle 2x+95 becomes

2(4)+95

8+95=103°

Therefore<3=180-105=75°

to train for a race, you plan to run 1 mile the first week and double the number of miles each week for five weeks. How many miles will you run for the 5th week. math problem

Answers

Answer:

16 Miles

Step-by-step explanation:

For every week you simply multiply the number of miles from the previous week by 2, therefore

Week 1: 1

Week 2: 2

Week 3: 4

Week 4: 8

Week 5: 16

A person walks 1/6 mile in 1/18 hour.

The person's speed is _ miles per hour.

Answers

This Is What I Got!

Hope This Helps! :)

Have A Good Day!!

And If You Can I Wouldn't Mind A Brainliest! :))

Answer:

Divide 1/6 miles to 1/12hour since u wanna find our miles per hour

So it’ll be : 1/6 / 1/12

= 1/6 x 12/1

= 2 miles

The ice cream man just ended his shift for the day. Let 1/2x^2 6/11x + 8 represent the amount of chocolate ice cream bars he sold. Let 5/9x^2 + 2/3 represent the amount of vanilla ice cream bars he sold. Finally let 1/3x^2 + 4x + 4/3 represent the amount of strawberry ice cream bars he sold. Select all the statements that are true

a. The total amount of ice cream bars sold can be represented by the expression 25/18x^2+ 50/11x +10
b. The total amount of ice cream bars sold can be represented by the expression 25/18x^2 + 172/33x +28/3
c. He sold 1/6x^2 + 50/11x + 28/3 more chocolate than strawberry ice cream bars.
d. He sold 1/6x^2 - 38/11x + 20/3 more chocolate than strawberry ice cream bars.

Answers

Answer:

A and D

Step-by-step explanation:

Total ice cream bars sold = sum of chocolate sold , vanilla and strawberry ice-creams sold.

=(1/2)x2 + (6/11)x + 8 + (5/9)x2 + (2/3) +(1/3)x2 + 4x +(4/3) (Given in the question)

=(25/18)x2 + (50/11)x + 10 (Adding terms corresponding to x2,x ,constant respectively)

Difference in chocolate and strawberry bars =[ (1/2)x2 + (6/11)x + 8] - [(1/3)x2 + 4x +(4/3)]

= (1/6)x2 - (38/11)x +(20/3)

So, the correct options are A and D

Find the area of the following shape:

Answers

Answer:

36cm^2

Step-by-step explanation:

total area: 6x(4+3)=42

total area excluding the space: 42-(2x3)=36

Answer:

36 cm squared

Step-by-step explanation:

To solve this problem, I first construct a line. (shown in yellow in the first photo)

I then find the area of the top rectangle. (6 cm * 4 cm = 24 cm squared.)

Next, I find the area of the lower rectangle. But...to do that I have to find the length of the line that I constructed. To do this, I do  6cm-2cm=4cm.

Then I can find the area of the lower rectangle. (4cm*3cm=12cm squared.)

add up the area of both of the rectangles and.........12+24=36 cm squared

If using the method of completing the square to solve the quadratic equation x^2+15x+21=0, which number would have to be added to "complete the square"?

Answers

Step-by-step explanation:

the answer is in the above image

Answer:

my answer is 225/5 sorry comments for wrong

21(2-y)+12y=44 find y​

Answers

Answer: y= -2/9
Explanation:
21(2-y)+12y=44
42-21y+12y=44
42-9y=44
-9y=2
y=-2/9

Answer:

[tex]\textbf{HELLO!!}[/tex]

[tex]21\left(2-y\right)+12y=44[/tex]

[tex]42-21y+12y=44[/tex]

[tex]~add ~similar\:elements[/tex]

[tex]42-9y=44[/tex]

[tex]Subtract~42~from~both~sides[/tex]

[tex]42-9y-42=44-42[/tex]

[tex]-9y=2[/tex]

[tex]Divide\:both\:sides\:by\:}-9[/tex]

[tex]\frac{-9y}{-9}=\frac{2}{-9}[/tex]

[tex]y=-\frac{2}{9}[/tex]

----------------------

hope it helps...

have a great day!

Suppose that from a group of 9 men, 1 will be randomly chosen for a dangerous assignment, and suppose that the chosen man will be killed during the assignment with a probability of 1/6. If Mark is one of the 9 men, what is the probability that he will be chosen for the assignment and killed during the assignment

Answers

Answer:

1/54

Step-by-step explanation:

1/9 x 1/6

Alec bakes spherical rolls of bread. Each roll is about 8cm
wide. What is the approximate volume of each roll? Use
3.14 to approximate a.

Answers

Answer:

Step-by-step explanation:

2143.57

0.7(1.5 + y) = 3.5y - 1.47

Answers

Answer:

y = 0.9

Step-by-step explanation:

1.05 + 0.7y = 3.5y - 1.47

-3.5y + 0.7y = -1.47 - 1.05

-2.8y = -2.52

y = 9/10 = 0.9

Answer:

[tex]\textbf{HELLO!!}[/tex]

[tex]0.7\left(1.5+y\right)=3.5y-1.47[/tex]

[tex]1.05+0.7y=3.5y-1.47 \gets \textsl{Expand}[/tex]

[tex]1.05+0.7y-1.05=3.5y-1.47-1.05 \gets Subtract\; 1.05 \from\:both\:sides[/tex]

[tex]0.7y=3.5y-2.52[/tex]

[tex]0.7y-3.5y=3.5y-2.52-3.5y[/tex]

[tex]\mathrm{Subtract\:}3.5y\mathrm{\:from\:both\:sides} \nwarrow[/tex]

[tex]-2.8y=-2.52[/tex]

[tex]\frac{-2.8y}{-2.8}=\frac{-2.52}{-2.8} \hookleftarrow \mathrm{Divide\:both\:sides\:by\:}-2.8[/tex]

[tex]\boxed{\boxed{\underline{\textsf{\textbf{y=0.9}}}}}[/tex]

[tex]\bullet \bullet \bullet \bullet \bullet \bullet \bullet \bullet \bullet \bullet \bullet \bullet \bullet[/tex]

[tex]\textbf{HOPE IT HELPS}[/tex]

[tex]\textbf{HAVE A GREAT DAY!!}[/tex]

Six consecutive numbers add up to a total of 69.what is the highest of these numbers?

Answers

Answer:

14

Step-by-step explanation:

9+10+11+12+13+14=69

The [tex]HIGHEST[/tex] of these numbers is [tex]14[/tex]

Let the [tex]CONSECUTIVE[/tex] numbers be : [tex]a, a+1, a +2,a+3,a+4,a+5[/tex]

Taking the sum :

[tex]a + a + 1 + a + 2 + a + 3 + a + 4 + a + 5 = 69\\6a + 15 = 69\\6a = 69 - 15\\6a = 54\\a = 54 / 6\\a = 9[/tex]

[tex]HIGHEST[/tex] value = [tex]a + 5 = 9 + 5 = 14[/tex]

Hence, [tex]HIGHEST[/tex] value = [tex]14[/tex]

Learn more : https://brainly.com/question/15974141

For this question I am sure the answer is 81% as you divide 45 and 55. However, it is stating my answer is incorrect even though I put 0.81% as well. Did I round wrong or is the answer wrong completely?

Answers

Answer:

it says round to the nearest 10th so it wouldn't be 81, it would be 81.8%

Moving to another question will save this response.
1 points
Save Answer
Question 12
Mr Espent 65% of his salary on household expenses, and 15% of the remainder on travelling expenses and was finally left with R9 500. How much was his salary?​

Answers

Answer:

rs.1680.67

Step-by-step explanation:

His salary = x

remaining % = 100 - 65 = 35%

= 100 - 15 = 85%

x × 35/100 × 85/100 = 500

x = 1680.67

B
These triangles
are congruent by
the triangle
congruence
postulate [?].
D
E
A. SSS
B. SAS
C. Neither, they are not congruent

Answers

Answer:

SAS

Step-by-step explanation:

AC ≅ EC (Given), ∠ACB ≅∠ECD ( Vertical Angles), and BC ≅ DC

he speeds (in MPH) of automobiles traveling in a city are given below:
20, 35, 42, 52, 65, 49, 24, 37, 23, 41, 50, 58
The mean speed of the cars is

Answers

Answer:

Mean speed = 41.3 mph

Step-by-step explanation:

Given that,

The speeds of an automobiles are given below:

20, 35, 42, 52, 65, 49, 24, 37, 23, 41, 50, 58

We need to find the mean speed of the cars.

Mean = sum of observations/ no. of observation

[tex]M=\dfrac{20+35+42+52+65+49+24+37+23+41+50+58}{12}\\\\M=41.3[/tex]

So, the mean speed of the cars is equal to 41.3 mph.

A road has a scale of 1:50 000 The length of a road on the map is 8.5cm.Work out the length of the real road in kilometres

Answers

Answer:

ok so

8.5*150000

1275000 cm into kilometers is

12.75 kilometers

Hope This Helps!!!

Solve the equation 2x^2 + 3 – 41 = –15 to the nearest tenth.

Hellllpppp

Answers

9514 1404 393

Answer:

 x = {-4.4, +2.9}

Step-by-step explanation:

We assume you want to solve ...

  2x^2 +3x -41 = -15

Adding 41 and factoring out the leading coefficient gives ...

  2(x^2 +3/2x) = 26

Dividing by 2 makes it ...

  x^2 +3/2x = 13

We can add the square of half the x-coefficient to "complete the square."

  x^2 +3/2x +(3/4)^2 = 13 +(3/4)^2

  (x +3/4)^2 = 13.5625 . . . . write the left side as a square

  x +3/4 = ±√13.5625 . . . . . take the square root

  x = -0.75 ±3.683 = {-4.433, +2.933} . . . . subtract 3/4 and evaluate

The solutions are approximately x = -4.4 and x = 2.9.

Joe drives for 3 hours and covers 201 miles. In miles per hour, how fast was he driving?​

Answers

Answer:

67 mph

Step-by-step explanation:

201/3 = 67

 Marsha has a bag that contains 4 green marbles, 8 yellow marbles , and 20 red marbles . If she chooses one marble from the bag, what is the probability that the marble is not red?

PLEASE HELP IF YOURE GOOD AT GEOMETRY!!

Answers

Answer:

C. 3/8

HOPE THIS HELPS :)

Answer:

c. 3/8

Step-by-step explanation:

first you need the denomerator by adding all marbles together which equals 32. now for the munerator you need the sum of the green and yellow marbles. this equals 12. so your fraction is 12/32. next we simplify. we can divide both numbers by 4. getting us a fraction of 3/8.

Stats question what are the main difference between frequenting and Bayesian

Answers

Answer:

The frequentist believes that probability represents long term frequencies of repeatable events (such a flipping a coin). Frequentists do not attach probabilities to hypotheses or unknown values. On the other hand, the Bayesian approach uses probabilities to represent the uncertainty in any event or hypothesis.

HOPE IT HELPS

Chung has 6 trucks and 5 cars in his toy box. Brian has 4 trucks and 5 cars in his toy box.

Which is the correct comparison of their ratios of trucks to cars?
StartFraction 6 Over 4 EndFraction less-than StartFraction 5 Over 5 EndFraction
StartFraction 6 Over 4 EndFraction greater-than StartFraction 5 Over 5 EndFraction
StartFraction 6 Over 5 EndFraction less-than StartFraction 4 Over 5 EndFraction
StartFraction 6 Over 5 EndFraction greater-than StartFraction 4 Over 5 EndFraction

Answers

Given:

Chung has 6 trucks and 5 cars in his toy box.

Brian has 4 trucks and 5 cars in his toy box.

To find:

The correct comparison of their ratios of trucks to cars.

Solution:

The ratio of trucks to cars is defined as:

[tex]\text{Ratio}=\dfrac{\text{Number of trucks}}{\text{Number of cars}}[/tex]

Chung has 6 trucks and 5 cars in his toy box. So, the ratio of trucks to cars is:

[tex]\text{Ratio}=\dfrac{6}{5}[/tex]

Brian has 4 trucks and 5 cars in his toy box.

[tex]\text{Ratio}=\dfrac{4}{5}[/tex]

We know that,

[tex]6>4[/tex]

[tex]\dfrac{6}{5}>\dfrac{4}{5}[/tex]

Therefore, the correct option is D.

Answer:

what the guy above me said

Step-by-step explanation:

so yeah he is right points

Can someone answer with steps and explanation? Thanks.

Answers

Answer:

[tex]x=-16\text{ or } x=7[/tex]

Step-by-step explanation:

Since ΔABC is mapped onto ΔDEF, we can write that:

[tex]\Delta ABC\cong \Delta DE F[/tex]

By CPCTC:

[tex]\angle A\cong \angle D[/tex]

And since ΔABC is isosceles with Vertex C:

[tex]\angle A \cong \angle B[/tex]

We are given that:

[tex]m\angle D=34[/tex]

Hence:

[tex]m\angle A=34=m\angle B[/tex]

We are also given that:

[tex]m\angle C=x^2+9x[/tex]

The interior angles of a triangle must sum to 180°. Thus:

[tex]m\angle A+m\angle B+m\angle C=180[/tex]

Substitute:

[tex](34)+(34)+(x^2+9x)=180[/tex]

Simplify:

[tex]68+x^2+9x=180[/tex]

Isolate the equation:

[tex]x^2+9x-112=0[/tex]

Factor:

[tex](x+16)(x-7)=0[/tex]

Zero Product Property:

[tex]x+16=0\text{ or } x-7=0[/tex]

Solve for each case:

[tex]x=-16\text{ or } x=7[/tex]

Testing the solutions, we can see that both yields C = 112°.

Hence, our solutions are:

[tex]x=-16\text{ or } x=7[/tex]

Other Questions
a) How much electrical energy, in joules, does a 1000W space heater consume when it runs for 8h? (b) How much energy is that in kilowatt hours? (c) Calculate the cost of using this heater if 1kWh costs 0.5 dollars. The name Bleeding Kansas came about as a result of _________________.a.violence between cattle ranchers and railroad buildersb.violence between pro-slavery and anti-slavery supportersc.violence between Native Americans and African Americansd.violence between Kansas and Nebraska residentsPlease select the best answer from the choices provided During the taking of its physical inventory on December 31, 2014, Barry's Bike Shop incorrectly counted its inventory as $229,134 instead of the correct amount of $165,639. The effect on the balance sheet and income statement would be An angle measuring 60 degrees is measured as 62 degrees. What is the percentage error correct to 3 significant figures? Find the measures of a positive angle and a negative angle that are coterminal with each given angle0=110 In the last six months, Sonia's family used 529, 499, 651, 652, 1,163, and 310 minutes on their cell phone plan. To save money, Sonia's family wants to keep their mean cell phone usage below 600 minutes per month.By how many minutes did they go over their goal in the last six months? Which set of numbers is arranged in increasing order? A. , , , B. , , , C. , , , D. , , , 25 points please explain it to me add the answer and the work since Im so confused on when to subtract missing lengths where and when what are historical thinking skills Mr. Shaes class is having a discussion about the writings of Shakespeare. One student says, "One theme found in Romeo and Juliet is that love can cause violence. Although the play is a famous love story, it is actually very violent.Which would be an appropriate follow-up question to ask this student?Who is the most romantic character in Shakespeares play Romeo and Juliet?What are some specific examples of love causing violence in Romeo and Juliet?Who were the most violent characters in this classic Shakespeare play?What are some symbols used by Shakespeare to depict love and violence? PLEASE HELP ME!! its a pretty easy question How did the Aboriginal people respond to European colonization of Australia in the 19th century?A. They sent diplomats to Europe to negotiate directly with powerful European leaders. B. They convinced the European settlers to turn their communities into protected reservations. C. They use their knowledge of local geography to launch attacks against European settlers. D. They integrated into European settlements to gain access to Superior European technology. After mixing the solutions in a separatory funnel, the stopper should be ______ and the liquid should be _______ and the layers allowed to separate. When you get close to the interface between the layers, ______ the funnel and turn over _______ heat up until the first layer is collected get eye level with to collect the second layer. _______ PLZZZ HELPFind the average rate of change of h(x) = 2x 7x from x=2 to x=5.Simplify your answer as much as possible. Kite Corporation has provided the following contribution format income statement. Assume that the following information is within the relevant range. Sales (3,000 units) $ 180,000 Variable expenses 108,000 Contribution margin 72,000 Fixed expenses 62,400 Net operating income $ 9,600The contribution margin ratio is closest to:______. a. 33% b. 40% c. 60% d. 67% [tex](2x + 3)(2 {x}^{2} - x - 2)[/tex]simplify this question Whose definition of justice left no room for moral interpretation?a-Cephalusb-Thrasymachusc-Polymarchusd-Adeimantus What is the ratio of the side length of the side opposite any 30 degree angle and the length of the hypothesis ? Why are the danzante figures such an important find?Fr help a brother out no links pls what phase of mitosis is shown in the diagram..? (Please can anyone help me out0